PT 29, Section 1 # 22 Forum

Prepare for the LSAT or discuss it with others in this forum.
Post Reply
pimpalicious1

New
Posts: 10
Joined: Sun Mar 21, 2010 3:27 am

PT 29, Section 1 # 22

Post by pimpalicious1 » Tue Aug 10, 2010 10:02 pm

Hey guys,

I feel totally brain dead right now and I just can't figure out exactly why answer choice D is the right answer. I eliminated all the wrong answers, but I am having trouble making out exactly what D is saying and why it correct. Any help would be greatly appreciated. Thanks!

User avatar
yzero1

Bronze
Posts: 185
Joined: Thu May 27, 2010 4:33 pm

Re: PT 29, Section 1 # 22

Post by yzero1 » Tue Aug 10, 2010 11:44 pm

In the stimulus, the government essentially makes contradictory claims.

Claim 1: Nuclear plants are entirely safe

Claim 2: Limiting the nuclear industry's financial liability is justified because it is necessary to protect the industry from bankrutcy

This second claim is then show to be inconsistent with the first using the following logical chain:

Unlimited liability a threat -> Injury claims can be sustained -> nuclear accident happens

So if nuclear accidents are completely safe, then via the contrapositive, unlimited liability is not a threat and limits are not necessary. The two claims cannot both be true.

The key is the author's conclusion: the public's fear is well founded. But how do we know which of the government's claims are correct? What if the nuclear power plants ARE safe, and the government's other claim about liability limitation is false?

In question 22, we look for something that will justify the editorial's conclusion that the public's fear of nuclear accidents is legitimate - in other words, the claim of safety is false.

(D) basically says that the government can make one claim, but its actions can reveal the truth of the claim. In this case, the unsupported claim is that nuclear power plants are safe.

You can also set up (D) as a conditional statement:

Act to prevent certain kind of situation from arising -> danger that such a situation will arise

So since the government HAS acted to prevent financial instability, then there must be a danger of bankruptcy, which means that there is a danger of a nuclear accident (we know this from the conditional chain built from the stimulus). Thus, the author's conclusion is justified.

Post Reply

Return to “LSAT Prep and Discussion Forum”